Q.1 [14 marks]#
Fill in the blanks/MCQs using appropriate choice from the given options.
Q1.1 [1 mark]#
$\begin{vmatrix} 5 & 1 \ 2 & 3 \end{vmatrix} = $ _______
Answer: b. 13
Solution: For 2×2 determinant $\begin{vmatrix} a & b \ c & d \end{vmatrix} = ad - bc$
$\begin{vmatrix} 5 & 1 \ 2 & 3 \end{vmatrix} = (5 \times 3) - (1 \times 2) = 15 - 2 = 13$
Q1.2 [1 mark]#
If $\begin{vmatrix} x & 1 \ 2 & 1 \end{vmatrix} = 0$ then $x = $ _______
Answer: b. 2
Solution: $\begin{vmatrix} x & 1 \ 2 & 1 \end{vmatrix} = x \times 1 - 1 \times 2 = x - 2 = 0$
Therefore, $x = 2$
Q1.3 [1 mark]#
If $f(x) = x^2$ then $f(-1) = $ _______
Answer: a. 1
Solution: $f(x) = x^2$ $f(-1) = (-1)^2 = 1$
Q1.4 [1 mark]#
$\log_{10} 1 = $ _______
Answer: b. 0
Solution: By logarithm property: $\log_a 1 = 0$ for any base $a > 0$ Therefore, $\log_{10} 1 = 0$
Q1.5 [1 mark]#
$\sin \frac{\pi}{2} + \cos \frac{\pi}{2} = $ _______
Answer: c. 1
Solution: $\sin \frac{\pi}{2} = 1$ and $\cos \frac{\pi}{2} = 0$ Therefore, $\sin \frac{\pi}{2} + \cos \frac{\pi}{2} = 1 + 0 = 1$
Q1.6 [1 mark]#
$\tan^{-1}(1) = $ _______
Answer: a. $\frac{\pi}{4}$
Solution: $\tan \frac{\pi}{4} = 1$ Therefore, $\tan^{-1}(1) = \frac{\pi}{4}$
Q1.7 [1 mark]#
$\frac{2\pi}{3}$ radian = _______ degree
Answer: d. 120
Solution: To convert radians to degrees: $\text{degrees} = \text{radians} \times \frac{180}{\pi}$ $\frac{2\pi}{3} \times \frac{180}{\pi} = \frac{2 \times 180}{3} = \frac{360}{3} = 120°$
Q1.8 [1 mark]#
$\hat{i} \times \hat{j} = $ _______
Answer: c. $\hat{k}$
Solution: By right-hand rule for cross product: $\hat{i} \times \hat{j} = \hat{k}$
Q1.9 [1 mark]#
$|\hat{i} + \hat{j} + \hat{k}| = $ _______
Answer: d. $\sqrt{3}$
Solution: $|\hat{i} + \hat{j} + \hat{k}| = \sqrt{1^2 + 1^2 + 1^2} = \sqrt{3}$
Q1.10 [1 mark]#
Slope of line $2x + y - 3 = 0$ is _______
Answer: a. -2
Solution: Convert to slope-intercept form: $y = -2x + 3$ Slope = coefficient of $x = -2$
Q1.11 [1 mark]#
Radius of circle $x^2 + y^2 = 81$ is _______
Answer: b. 9
Solution: Standard form: $x^2 + y^2 = r^2$ Here, $r^2 = 81$, so $r = 9$
Q1.12 [1 mark]#
$\lim_{n \to \infty} \frac{1}{n} = $ _______
Answer: c. 0
Solution: As $n$ approaches infinity, $\frac{1}{n}$ approaches 0
Q1.13 [1 mark]#
$\lim_{x \to 1} (x^2 + x + 1) = $ _______
Answer: a. 3
Solution: Direct substitution: $(1)^2 + (1) + 1 = 1 + 1 + 1 = 3$
Q1.14 [1 mark]#
$\lim_{\theta \to 0} \frac{\tan \theta}{\theta} = $ _______
Answer: b. 1
Solution: This is a standard limit: $\lim_{\theta \to 0} \frac{\tan \theta}{\theta} = 1$
Q.2 (A) [6 marks]#
Attempt any two
Q2.1 [3 marks]#
Find the value of $\begin{vmatrix} 1 & 3 & 1 \ 2 & -1 & 0 \ 4 & -2 & 5 \end{vmatrix}$
Answer:
Solution: Using expansion along second row (has zero): $= -2\begin{vmatrix} 3 & 1 \ -2 & 5 \end{vmatrix} + (-1)\begin{vmatrix} 1 & 1 \ 4 & 5 \end{vmatrix} + 0$
$= -2(15 + 2) - 1(5 - 4)$ $= -2(17) - 1(1)$ $= -34 - 1 = -35$
Table:
Step | Calculation | Result |
---|---|---|
Minor 1 | $(3 \times 5) - (1 \times -2)$ | 17 |
Minor 2 | $(1 \times 5) - (1 \times 4)$ | 1 |
Final | $-2(17) - 1(1)$ | -35 |
Q2.2 [3 marks]#
If $f(x) = x^3 + 5$ then find $f(0)$, $f(1)$ and $f(-1)$
Answer:
Solution: Given: $f(x) = x^3 + 5$
$f(0) = (0)^3 + 5 = 0 + 5 = 5$ $f(1) = (1)^3 + 5 = 1 + 5 = 6$ $f(-1) = (-1)^3 + 5 = -1 + 5 = 4$
Table:
Input | Calculation | Output |
---|---|---|
$f(0)$ | $0^3 + 5$ | 5 |
$f(1)$ | $1^3 + 5$ | 6 |
$f(-1)$ | $(-1)^3 + 5$ | 4 |
Q2.3 [3 marks]#
Prove that $\tan^{-1}\left(\frac{1}{2}\right) + \tan^{-1}\left(\frac{1}{3}\right) = \frac{\pi}{4}$
Answer:
Solution: Using formula: $\tan^{-1}a + \tan^{-1}b = \tan^{-1}\left(\frac{a+b}{1-ab}\right)$
Let $a = \frac{1}{2}$, $b = \frac{1}{3}$
$\tan^{-1}\left(\frac{1}{2}\right) + \tan^{-1}\left(\frac{1}{3}\right) = \tan^{-1}\left(\frac{\frac{1}{2} + \frac{1}{3}}{1 - \frac{1}{2} \times \frac{1}{3}}\right)$
$= \tan^{-1}\left(\frac{\frac{5}{6}}{1 - \frac{1}{6}}\right) = \tan^{-1}\left(\frac{\frac{5}{6}}{\frac{5}{6}}\right) = \tan^{-1}(1) = \frac{\pi}{4}$
Hence proved.
Q.2 (B) [8 marks]#
Attempt any two
Q2.1 [4 marks]#
If $f(x) = \frac{x-1}{x+1}$ then prove that $f(x) \cdot f(-x) = 1$
Answer:
Solution: Given: $f(x) = \frac{x-1}{x+1}$
First find $f(-x)$: $f(-x) = \frac{(-x)-1}{(-x)+1} = \frac{-x-1}{-x+1} = \frac{-(x+1)}{-(x-1)} = \frac{x+1}{x-1}$
Now calculate $f(x) \cdot f(-x)$: $f(x) \cdot f(-x) = \frac{x-1}{x+1} \cdot \frac{x+1}{x-1} = \frac{(x-1)(x+1)}{(x+1)(x-1)} = 1$
Hence proved.
Q2.2 [4 marks]#
If $\log\left(\frac{x+y}{2}\right) = \frac{1}{2}(\log x + \log y)$ then prove that $x = y$
Answer:
Solution: Given: $\log\left(\frac{x+y}{2}\right) = \frac{1}{2}(\log x + \log y)$
Using logarithm properties: $\frac{1}{2}(\log x + \log y) = \frac{1}{2}\log(xy) = \log\sqrt{xy}$
So: $\log\left(\frac{x+y}{2}\right) = \log\sqrt{xy}$
Taking antilog: $\frac{x+y}{2} = \sqrt{xy}$
Squaring both sides: $\left(\frac{x+y}{2}\right)^2 = xy$
$\frac{(x+y)^2}{4} = xy$
$(x+y)^2 = 4xy$
$x^2 + 2xy + y^2 = 4xy$
$x^2 - 2xy + y^2 = 0$
$(x-y)^2 = 0$
Therefore, $x = y$. Hence proved.
Q2.3 [4 marks]#
Solve $\log(x+3) + \log(x-3) = \log 27$
Answer:
Solution: Given: $\log(x+3) + \log(x-3) = \log 27$
Using logarithm property: $\log a + \log b = \log(ab)$ $\log[(x+3)(x-3)] = \log 27$
Taking antilog: $(x+3)(x-3) = 27$
$x^2 - 9 = 27$
$x^2 = 36$
$x = \pm 6$
Check validity:
- For $x = 6$: $x+3 = 9 > 0$ and $x-3 = 3 > 0$ ✓
- For $x = -6$: $x+3 = -3 < 0$ (invalid for logarithm)
Therefore, $x = 6$
Q.3 (A) [6 marks]#
Attempt any two
Q3.1 [3 marks]#
Prove that $\frac{\sin\left(\frac{\pi}{2}+\theta\right)}{\cos(\pi-\theta)} + \frac{\tan(\pi-\theta)}{\cot\left(\frac{3\pi}{2}-\theta\right)} + \frac{\text{cosec}\left(\frac{\pi}{2}-\theta\right)}{\sec(\pi+\theta)} = -3$
Answer:
Solution: Using trigonometric identities:
$\sin\left(\frac{\pi}{2}+\theta\right) = \cos\theta$ $\cos(\pi-\theta) = -\cos\theta$ $\tan(\pi-\theta) = -\tan\theta$ $\cot\left(\frac{3\pi}{2}-\theta\right) = \tan\theta$ $\text{cosec}\left(\frac{\pi}{2}-\theta\right) = \sec\theta$ $\sec(\pi+\theta) = -\sec\theta$
Substituting: $\frac{\cos\theta}{-\cos\theta} + \frac{-\tan\theta}{\tan\theta} + \frac{\sec\theta}{-\sec\theta}$
$= -1 + (-1) + (-1) = -3$
Hence proved.
Q3.2 [3 marks]#
Prove that $\tan 55° = \frac{\cos 10° + \sin 10°}{\cos 10° - \sin 10°}$
Answer:
Solution: We know that $\tan 55° = \tan(45° + 10°)$
Using formula: $\tan(A + B) = \frac{\tan A + \tan B}{1 - \tan A \tan B}$
$\tan 55° = \frac{\tan 45° + \tan 10°}{1 - \tan 45° \tan 10°} = \frac{1 + \tan 10°}{1 - \tan 10°}$
Now, $\tan 10° = \frac{\sin 10°}{\cos 10°}$
$\tan 55° = \frac{1 + \frac{\sin 10°}{\cos 10°}}{1 - \frac{\sin 10°}{\cos 10°}} = \frac{\cos 10° + \sin 10°}{\cos 10° - \sin 10°}$
Hence proved.
Q3.3 [3 marks]#
If $\vec{a} = 2\hat{i} + 3\hat{j} + \hat{k}$, $\vec{b} = \hat{i} + \hat{j} + \hat{k}$ and $\vec{c} = 3\hat{i} + \hat{j} + \hat{k}$ then find $2\vec{a} + \vec{b} - \vec{c}$
Answer:
Solution: Given: $\vec{a} = 2\hat{i} + 3\hat{j} + \hat{k}$ $\vec{b} = \hat{i} + \hat{j} + \hat{k}$ $\vec{c} = 3\hat{i} + \hat{j} + \hat{k}$
$2\vec{a} = 2(2\hat{i} + 3\hat{j} + \hat{k}) = 4\hat{i} + 6\hat{j} + 2\hat{k}$
$2\vec{a} + \vec{b} - \vec{c} = (4\hat{i} + 6\hat{j} + 2\hat{k}) + (\hat{i} + \hat{j} + \hat{k}) - (3\hat{i} + \hat{j} + \hat{k})$
$= (4 + 1 - 3)\hat{i} + (6 + 1 - 1)\hat{j} + (2 + 1 - 1)\hat{k}$
$= 2\hat{i} + 6\hat{j} + 2\hat{k}$
Q.3 (B) [8 marks]#
Attempt any two
Q3.1 [4 marks]#
Prove that $\frac{\sin(x-y)}{\cos x \cos y} + \frac{\sin(y-z)}{\cos y \cos z} + \frac{\sin(z-x)}{\cos z \cos x} = 0$
Answer:
Solution: Using identity: $\sin(A-B) = \sin A \cos B - \cos A \sin B$
$\frac{\sin(x-y)}{\cos x \cos y} = \frac{\sin x \cos y - \cos x \sin y}{\cos x \cos y} = \tan x - \tan y$
Similarly: $\frac{\sin(y-z)}{\cos y \cos z} = \tan y - \tan z$ $\frac{\sin(z-x)}{\cos z \cos x} = \tan z - \tan x$
Adding all three: $(\tan x - \tan y) + (\tan y - \tan z) + (\tan z - \tan x) = 0$
Hence proved.
Q3.2 [4 marks]#
Draw graph of $y = \cos x$ for $0 \leq x \leq \pi$
Answer:
Solution:
Table of values:
x | 0 | π/4 | π/2 | 3π/4 | π |
---|---|---|---|---|---|
y | 1 | √2/2 | 0 | -√2/2 | -1 |
Q3.3 [4 marks]#
Find equation of line passing through (1, 2) and (-3, 1)
Answer:
Solution: Given points: $(x_1, y_1) = (1, 2)$ and $(x_2, y_2) = (-3, 1)$
Slope: $m = \frac{y_2 - y_1}{x_2 - x_1} = \frac{1 - 2}{-3 - 1} = \frac{-1}{-4} = \frac{1}{4}$
Using point-slope form: $y - y_1 = m(x - x_1)$ $y - 2 = \frac{1}{4}(x - 1)$ $4(y - 2) = x - 1$ $4y - 8 = x - 1$ $x - 4y + 7 = 0$
Equation: $x - 4y + 7 = 0$
Q.4 (A) [6 marks]#
Attempt any two
Q4.1 [3 marks]#
Find unit vector perpendicular to $\vec{a} = \hat{i} - 3\hat{j} + \hat{k}$ and $\vec{b} = 2\hat{i} + \hat{j} + 2\hat{k}$
Answer:
Solution: Cross product: $\vec{a} \times \vec{b} = \begin{vmatrix} \hat{i} & \hat{j} & \hat{k} \ 1 & -3 & 1 \ 2 & 1 & 2 \end{vmatrix}$
$= \hat{i}[(-3)(2) - (1)(1)] - \hat{j}[(1)(2) - (1)(2)] + \hat{k}[(1)(1) - (-3)(2)]$ $= \hat{i}(-6-1) - \hat{j}(2-2) + \hat{k}(1+6)$ $= -7\hat{i} + 0\hat{j} + 7\hat{k}$
Magnitude: $|\vec{a} \times \vec{b}| = \sqrt{(-7)^2 + 0^2 + 7^2} = \sqrt{49 + 49} = 7\sqrt{2}$
Unit vector: $\hat{n} = \frac{-7\hat{i} + 7\hat{k}}{7\sqrt{2}} = \frac{-\hat{i} + \hat{k}}{\sqrt{2}}$
Q4.2 [3 marks]#
Forces (1, 2, 1) and (2, -1, 3) act on a particle and the particle moves from point (2, 3, 1) to (4, 6, 2). Find the work done.
Answer:
Solution: Resultant force: $\vec{F} = (1, 2, 1) + (2, -1, 3) = (3, 1, 4)$
Displacement: $\vec{s} = (4, 6, 2) - (2, 3, 1) = (2, 3, 1)$
Work done: $W = \vec{F} \cdot \vec{s} = (3)(2) + (1)(3) + (4)(1) = 6 + 3 + 4 = 13$ units
Q4.3 [3 marks]#
Show that lines $2x - 3y + 5 = 0$ and $8x - 12y - 3 = 0$ are parallel lines.
Answer:
Solution: For line $2x - 3y + 5 = 0$: slope $m_1 = \frac{2}{3}$ For line $8x - 12y - 3 = 0$: slope $m_2 = \frac{8}{12} = \frac{2}{3}$
Since $m_1 = m_2 = \frac{2}{3}$, the lines are parallel.
Table:
Line | Standard Form | Slope |
---|---|---|
Line 1 | $2x - 3y + 5 = 0$ | $\frac{2}{3}$ |
Line 2 | $8x - 12y - 3 = 0$ | $\frac{2}{3}$ |
Q.4 (B) [8 marks]#
Attempt any two
Q4.1 [4 marks]#
Show that angle between $\vec{a} = \hat{i} + \hat{j} - \hat{k}$ and $\vec{b} = 2\hat{i} - 2\hat{j} + \hat{k}$ is $\sin^{-1}\left(\frac{\sqrt{26}}{27}\right)$
Answer:
Solution: $\vec{a} \cdot \vec{b} = (1)(2) + (1)(-2) + (-1)(1) = 2 - 2 - 1 = -1$
$|\vec{a}| = \sqrt{1^2 + 1^2 + (-1)^2} = \sqrt{3}$ $|\vec{b}| = \sqrt{2^2 + (-2)^2 + 1^2} = \sqrt{9} = 3$
$\cos\theta = \frac{\vec{a} \cdot \vec{b}}{|\vec{a}||\vec{b}|} = \frac{-1}{\sqrt{3} \times 3} = \frac{-1}{3\sqrt{3}}$
$\sin^2\theta = 1 - \cos^2\theta = 1 - \frac{1}{27} = \frac{26}{27}$
Therefore, $\sin\theta = \frac{\sqrt{26}}{3\sqrt{3}} = \frac{\sqrt{26}}{\sqrt{27}}$
Hence, $\theta = \sin^{-1}\left(\frac{\sqrt{26}}{\sqrt{27}}\right)$
Q4.2 [4 marks]#
If $\vec{a} = (1, 1, 1)$, $\vec{b} = (2, 0, 1)$ and $\vec{c} = (-2, 1, 0)$ then find $\vec{a} \cdot (\vec{b} \times \vec{c})$
Answer:
Solution: First find $\vec{b} \times \vec{c}$: $\vec{b} \times \vec{c} = \begin{vmatrix} \hat{i} & \hat{j} & \hat{k} \ 2 & 0 & 1 \ -2 & 1 & 0 \end{vmatrix}$
$= \hat{i}(0 \times 0 - 1 \times 1) - \hat{j}(2 \times 0 - 1 \times (-2)) + \hat{k}(2 \times 1 - 0 \times (-2))$ $= \hat{i}(-1) - \hat{j}(2) + \hat{k}(2)$ $= -\hat{i} - 2\hat{j} + 2\hat{k}$
Now find $\vec{a} \cdot (\vec{b} \times \vec{c})$: $\vec{a} \cdot (\vec{b} \times \vec{c}) = (1, 1, 1) \cdot (-1, -2, 2)$ $= (1)(-1) + (1)(-2) + (1)(2) = -1 - 2 + 2 = -1$
Q4.3 [4 marks]#
Evaluate $\lim_{\theta \to 0} \frac{\sin 4\theta}{\theta}$
Answer:
Solution: $\lim_{\theta \to 0} \frac{\sin 4\theta}{\theta} = \lim_{\theta \to 0} \frac{\sin 4\theta}{4\theta} \times 4$
Using standard limit $\lim_{x \to 0} \frac{\sin x}{x} = 1$:
Let $u = 4\theta$, then as $\theta \to 0$, $u \to 0$
$\lim_{\theta \to 0} \frac{\sin 4\theta}{4\theta} = \lim_{u \to 0} \frac{\sin u}{u} = 1$
Therefore, $\lim_{\theta \to 0} \frac{\sin 4\theta}{\theta} = 4 \times 1 = 4$
Q.5 (A) [6 marks]#
Attempt any two
Q5.1 [3 marks]#
Evaluate $\lim_{x \to 9} \frac{x^2 - 81}{x - 9}$
Answer:
Solution: Direct substitution gives $\frac{0}{0}$ form.
Factor the numerator: $x^2 - 81 = (x-9)(x+9)$
$\lim_{x \to 9} \frac{x^2 - 81}{x - 9} = \lim_{x \to 9} \frac{(x-9)(x+9)}{x-9}$
$= \lim_{x \to 9} (x+9) = 9 + 9 = 18$
Q5.2 [3 marks]#
Evaluate $\lim_{x \to \infty} \left(1 + \frac{3}{x}\right)^{2x}$
Answer:
Solution: Let $y = \left(1 + \frac{3}{x}\right)^{2x}$
Taking natural logarithm: $\ln y = 2x \ln\left(1 + \frac{3}{x}\right)$
As $x \to \infty$, $\frac{3}{x} \to 0$
Using $\ln(1+u) \approx u$ for small $u$: $\ln y = 2x \times \frac{3}{x} = 6$
Therefore, $y = e^6$
Q5.3 [3 marks]#
Evaluate $\lim_{x \to 1} \frac{x - 1}{x^2 + x - 2}$
Answer:
Solution: Factor the denominator: $x^2 + x - 2 = (x+2)(x-1)$
$\lim_{x \to 1} \frac{x - 1}{x^2 + x - 2} = \lim_{x \to 1} \frac{x-1}{(x+2)(x-1)}$
$= \lim_{x \to 1} \frac{1}{x+2} = \frac{1}{1+2} = \frac{1}{3}$
Q.5 (B) [8 marks]#
Attempt any two
Q5.1 [4 marks]#
Find the equation of line passing through the point (2, -3) and having slope 4.
Answer:
Solution: Using point-slope form: $y - y_1 = m(x - x_1)$
Given: $(x_1, y_1) = (2, -3)$ and slope $m = 4$
$y - (-3) = 4(x - 2)$ $y + 3 = 4x - 8$ $y = 4x - 11$
Equation: $y = 4x - 11$ or $4x - y - 11 = 0
Q5.2 [4 marks]#
For what value of m, lines $7x + y - 1 = 0$ and $3x - my + 2 = 0$ are perpendicular to each other.
Answer:
Solution: For perpendicular lines, product of slopes = -1
For line $7x + y - 1 = 0$: slope $m_1 = -7$ For line $3x - my + 2 = 0$: slope $m_2 = \frac{3}{m}$
Condition: $m_1 \times m_2 = -1$ $(-7) \times \frac{3}{m} = -1$ $\frac{-21}{m} = -1$ $21 = m$
Therefore, $m = 21$
Table:
Line | Standard Form | Slope |
---|---|---|
Line 1 | $7x + y - 1 = 0$ | $-7$ |
Line 2 | $3x - my + 2 = 0$ | $\frac{3}{m}$ |
Verification: When $m = 21$, slopes are $-7$ and $\frac{3}{21} = \frac{1}{7}$ Product: $(-7) \times \frac{1}{7} = -1$ ✓
Q5.3 [4 marks]#
Find the centre and radius of the circle $4x^2 + 4y^2 + 8x - 12y - 3 = 0$
Answer:
Solution: First, divide by 4 to get standard form: $x^2 + y^2 + 2x - 3y - \frac{3}{4} = 0$
Complete the square for x and y terms: $x^2 + 2x = (x+1)^2 - 1$ $y^2 - 3y = \left(y - \frac{3}{2}\right)^2 - \frac{9}{4}$
Substituting: $(x+1)^2 - 1 + \left(y - \frac{3}{2}\right)^2 - \frac{9}{4} - \frac{3}{4} = 0$
$(x+1)^2 + \left(y - \frac{3}{2}\right)^2 = 1 + \frac{9}{4} + \frac{3}{4} = 1 + 3 = 4$
Centre: $(-1, \frac{3}{2})$ Radius: $r = \sqrt{4} = 2$
Table:
Component | Value |
---|---|
Centre (h,k) | $(-1, \frac{3}{2})$ |
Radius | 2 |
Standard Form | $(x+1)^2 + (y-\frac{3}{2})^2 = 4$ |
Formula Cheat Sheet#
Determinants#
- 2×2 Determinant: $\begin{vmatrix} a & b \ c & d \end{vmatrix} = ad - bc$
- 3×3 Determinant: Expand along any row/column
Functions & Logarithms#
- Basic: $\log_a 1 = 0$, $\log_a a = 1$
- Properties: $\log(ab) = \log a + \log b$, $\log\left(\frac{a}{b}\right) = \log a - \log b$
Trigonometry#
- Basic Values: $\sin 0° = 0$, $\sin 30° = \frac{1}{2}$, $\sin 45° = \frac{\sqrt{2}}{2}$, $\sin 60° = \frac{\sqrt{3}}{2}$, $\sin 90° = 1$
- Conversion: Radians to degrees: $\times \frac{180}{\pi}$
- Identities: $\sin^2\theta + \cos^2\theta = 1$
- Inverse: $\tan^{-1}(1) = \frac{\pi}{4}$
Vectors#
- Magnitude: $|\vec{a}| = \sqrt{a_x^2 + a_y^2 + a_z^2}$
- Dot Product: $\vec{a} \cdot \vec{b} = a_x b_x + a_y b_y + a_z b_z$
- Cross Product: $\hat{i} \times \hat{j} = \hat{k}$, $\hat{j} \times \hat{k} = \hat{i}$, $\hat{k} \times \hat{i} = \hat{j}$
- Work Done: $W = \vec{F} \cdot \vec{s}$
Coordinate Geometry#
- Slope: $m = \frac{y_2 - y_1}{x_2 - x_1}$
- Point-Slope Form: $y - y_1 = m(x - x_1)$
- Parallel Lines: Same slope
- Perpendicular Lines: Product of slopes = -1
- Circle: $(x-h)^2 + (y-k)^2 = r^2$
Limits#
- Standard Limits: $\lim_{x \to 0} \frac{\sin x}{x} = 1$, $\lim_{x \to 0} \frac{\tan x}{x} = 1$
- Factorization: Use for $\frac{0}{0}$ forms
- L’Hôpital’s Rule: For indeterminate forms
Problem-Solving Strategies#
For Determinants:#
- Choose the row/column with most zeros for expansion
- Use cofactor expansion systematically
- Check calculations by expanding along different rows
For Functions:#
- Direct substitution first
- Use function properties and definitions
- Check domain restrictions
For Trigonometry:#
- Convert all angles to same unit (degrees or radians)
- Use standard angle values
- Apply appropriate identities
- Simplify step by step
For Vectors:#
- Write components clearly
- Use right-hand rule for cross products
- Check units and directions
- Verify with geometric interpretation
For Coordinate Geometry:#
- Plot points when possible
- Use appropriate formulas based on given information
- Check parallel/perpendicular conditions
- Complete the square for circles
For Limits:#
- Try direct substitution first
- Factor polynomials for $\frac{0}{0}$ forms
- Use standard limit formulas
- Apply L’Hôpital’s rule for indeterminate forms
Common Mistakes to Avoid#
Determinants:#
- ❌ Wrong sign in calculations
- ✅ Follow cofactor signs carefully: $(-1)^{i+j}$
Logarithms:#
- ❌ $\log(a+b) = \log a + \log b$ (WRONG)
- ✅ $\log(ab) = \log a + \log b$ (CORRECT)
Trigonometry:#
- ❌ Mixing degrees and radians
- ✅ Convert to same unit first
Vectors:#
- ❌ $\vec{a} \times \vec{b} = \vec{b} \times \vec{a}$ (WRONG)
- ✅ $\vec{a} \times \vec{b} = -(\vec{b} \times \vec{a})$ (CORRECT)
Slopes:#
- ❌ Confusing parallel and perpendicular conditions
- ✅ Parallel: same slope, Perpendicular: product = -1
Limits:#
- ❌ Direct substitution without checking indeterminate forms
- ✅ Check for $\frac{0}{0}$ or $\frac{\infty}{\infty}$ first
Exam Tips#
Time Management:#
- Spend 2 minutes per mark (14 marks = 28 minutes for Q1)
- Start with familiar questions
- Leave difficult problems for the end
Calculation Tips:#
- Show all steps clearly
- Use tables for organized presentation
- Double-check arithmetic
- Write final answers clearly
Writing Strategy:#
- Write given information first
- State formulas before using them
- Include units where applicable
- Box or underline final answers
Last-Minute Checks:#
- Verify all calculations
- Check if answers are reasonable
- Ensure all parts are attempted
- Review question requirements
Mnemonic for Standard Angles: “Some People Have Curly Brown Hair Through Proper Brushing”
- Sin 0° = 0, Pi/6 = 1/2, Half = √2/2, Cos complement, etc.
Remember: Mathematics is about understanding patterns, not memorizing formulas. Practice regularly and think step by step!
Quick Reference Table#
Topic | Key Formula | Example |
---|---|---|
Determinant 2×2 | $ad - bc$ | $\begin{vmatrix} 2 & 3 \ 1 & 4 \end{vmatrix} = 8-3 = 5$ |
Slope | $\frac{y_2-y_1}{x_2-x_1}$ | Points (1,2), (3,8): $m = \frac{8-2}{3-1} = 3$ |
Distance | $\sqrt{(x_2-x_1)^2+(y_2-y_1)^2}$ | Between (0,0), (3,4): $d = 5$ |
Circle | $(x-h)^2+(y-k)^2=r^2$ | Center (1,2), radius 3 |
Limit | $\lim_{x \to a} f(x)$ | Direct substitution or factoring |
Final Tip: Keep practicing and stay confident! 🎯